caspiata di fisica

Vuoi proporre i tuoi esercizi? Qui puoi farlo!!

Moderatore: tutor

germania2002
Messaggi: 821
Iscritto il: 01 gen 1970, 01:00
Località: Cosenza
Contatta:

Messaggio da germania2002 »

ah ok.....ora questo
<BR>
<BR><!-- BBCode Start --><B>nuovo problema</B><!-- BBCode End -->
<BR>
<BR>Io sono riuscito a calcolare 1 caso su 3, però ad intuito e l\'ho fatto l\'anno scorso, magari se applico la matematica mi vengono anche gli altri 2.
<BR>
<BR>Sempre dal libro fisica 1.
<BR>
<BR>(preso dalla rivista ungherese komal 1994) Due ragazzi si allenano in piscina: si tuffano insieme dagli estremi opposti della vasca e procedono a velocità costante; giunti in fondo, invertono il percorso e continuano a nuotare, ciascuno sempre con la propria velocità costante iniziale. Il primo incontro dei due avviene a 22metri dall\'estremo sud della vasca e il secondo a 16 metri dall\'estremo nord. Quanto può essere lunga la vasca?
<BR>
<BR>3 possibili soluzioni.[addsig]
"un uomo deve migliorare di qualcosa il mondo, se si vuole sentire realizzato..."
"Deutschland der beste Staat!"
[url:pvcj9bic]http://www.grid.org[/url:pvcj9bic] (pc vs cancro,sars,peste)
mola6
Messaggi: 220
Iscritto il: 01 gen 1970, 01:00
Località: Rimini

Messaggio da mola6 »

una mi viene 50mt. è giusta?[addsig]
"Per perdere la testa, bisogna innanzi tutto averne una!" A. Einstein
germania2002
Messaggi: 821
Iscritto il: 01 gen 1970, 01:00
Località: Cosenza
Contatta:

Messaggio da germania2002 »

e quella che avevo trovato anch\'io, ma ce ne sono altre 2.
<BR>cmq è giusta[addsig]
"un uomo deve migliorare di qualcosa il mondo, se si vuole sentire realizzato..."
"Deutschland der beste Staat!"
[url:pvcj9bic]http://www.grid.org[/url:pvcj9bic] (pc vs cancro,sars,peste)
mola6
Messaggi: 220
Iscritto il: 01 gen 1970, 01:00
Località: Rimini

Messaggio da mola6 »

ma facendo il sistema mi viene solo questa soluzione:
<BR>
<BR>l= lung vasca
<BR>v= veloc del nuotat da N
<BR>w= veloc del nuot da S
<BR>t= tempo del primo incontro
<BR>T= t del secondo
<BR>
<BR>2l-16=vT
<BR>l+16=wT
<BR>l-22=vt
<BR>22=wt
<BR>
<BR>T=3t
<BR>
<BR>e così mi viene solo 50mt. le altre?
<BR>
<BR>per perdere la testa, bisogna innanzi tutto averne una<BR><BR>[ Questo Messaggio è stato Modificato da: mola6 il 31-12-2003 11:28 ]
"Per perdere la testa, bisogna innanzi tutto averne una!" A. Einstein
germania2002
Messaggi: 821
Iscritto il: 01 gen 1970, 01:00
Località: Cosenza
Contatta:

Messaggio da germania2002 »

le altre sono 30 e 72, ma non ci arrivo.........[addsig]
"un uomo deve migliorare di qualcosa il mondo, se si vuole sentire realizzato..."
"Deutschland der beste Staat!"
[url:pvcj9bic]http://www.grid.org[/url:pvcj9bic] (pc vs cancro,sars,peste)
mola6
Messaggi: 220
Iscritto il: 01 gen 1970, 01:00
Località: Rimini

Messaggio da mola6 »

sì a 30 c\'ero arrivato anch\'io, ma per tentativi... [addsig]
"Per perdere la testa, bisogna innanzi tutto averne una!" A. Einstein
mola6
Messaggi: 220
Iscritto il: 01 gen 1970, 01:00
Località: Rimini

Messaggio da mola6 »

beh? non lo fa nessuno il problema?
"Per perdere la testa, bisogna innanzi tutto averne una!" A. Einstein
ma_go
Site Admin
Messaggi: 1906
Iscritto il: 01 gen 1970, 01:00

Messaggio da ma_go »

io vi butto lì una domanda:
<BR>chi vi dice che il secondo incontro sia ancora \"incrociato\"? o, in altre parole, chi vi dice che il secondo incontro avvenga mentre ciascuno dei due sta facendo la propria seconda vasca?
germania2002
Messaggi: 821
Iscritto il: 01 gen 1970, 01:00
Località: Cosenza
Contatta:

Messaggio da germania2002 »

mumble, interessante ma_go..non c\'avevo pensato, in teoria uno viaggerebbe ad una velocità astronomica ripetto l\'altro, wow......
<BR>
<BR>Cmq:
<BR>
<BR><!-- BBCode Start --><B>Problema 1</B><!-- BBCode End -->
<BR>
<BR>Sempre dal libro fisica 1.
<BR>
<BR>(preso dalla rivista ungherese komal 1994) Due ragazzi si allenano in piscina: si tuffano insieme dagli estremi opposti della vasca e procedono a velocità costante; giunti in fondo, invertono il percorso e continuano a nuotare, ciascuno sempre con la propria velocità costante iniziale. Il primo incontro dei due avviene a 22metri dall\'estremo sud della vasca e il secondo a 16 metri dall\'estremo nord. Quanto può essere lunga la vasca?
<BR>
<BR>3 possibili soluzioni.
<BR>[30,50,72m)
<BR>
<BR>MOLA6
<BR>
<BR>l= lung vasca
<BR>v= veloc del nuotat da N
<BR>w= veloc del nuot da S
<BR>t= tempo del primo incontro
<BR>T= t del secondo
<BR>
<BR>2l-16=vT
<BR>l+16=wT
<BR>l-22=vt
<BR>22=wt
<BR>
<BR>T=3t
<BR>
<BR>e così mi viene solo 50mt.
<BR>
<BR><!-- BBCode Start --><B>Problema 2</B><!-- BBCode End -->
<BR>
<BR>sempre da libro di fisica 1.
<BR>
<BR>In un fiume di larghezza h si può grossolanamente assumere che la velocità dell\'acqua aumenti proporzionalmente alla distanza delle sponde e raggiunga il valore massimo Vacqua0 (Va0) nel centro del fiume. In prossimità delle due sponde la velocità dell\'acqua è nulla. Un\'imbarcazione si stà muovendo da una sponda all\'altra con una velocità Vbarca (Vb), relativa all\'acqua che è costante e perpendicolare alla corrente fluviale.
<BR>1) Determina di quanto la corrente sposta la barca che attravera il fiume
<BR>2) Determina, inoltre, la traiettoria dell\'imbarcazione.
<BR>RISOLVERE: con le equazioni dei moti uniformi ed uniformemente accelerati, con la composizione dei moti e con la geometria.
<BR>
<BR>Visto che questo problema l\'ho iniziato ieri, vorrei scrivere la mia soluzione perchè io il 99.9....9% delle volte sparo cazzate.
<BR>
<BR>1)Ho notato che la Vb è relativa alla Va0, quindi ci deve essere un rapporto che rimane costante. Quindi Vb/Va0= kost.
<BR>Detto questo ho pensato di applicare la proporzione alle velocità e allo spostamento:
<BR>
<BR>Vb : Va0 = h/2 : s (1)
<BR>
<BR>dove s è lo spostamento che si deve calcolare e h/2 è il punto dove la velocità è maggiore.
<BR>Dalla (1) posso ricavare:
<BR>
<BR>s= (Va0*h)/(2Vb)
<BR>
<BR>è giusto????
<BR>
<BR>2) non la dico perchè credo sia facile.
<BR>
<BR>GRAZIE PER LE EVENTUALI CORREZIONI che mi farete notare.
<BR>
<BR>\"un uomo deve migliorare di qualcosa il mondo, se si vuole sentire realizzato...\"
<BR>\"Deutschland der beste Staat!\"
<BR><!-- BBCode Start --><A HREF="http://www.grid.org" TARGET="_blank">www.grid.org</A><!-- BBCode End --> (pc vs cancro,sars,peste)<BR><BR>[ Questo Messaggio è stato Modificato da: germania2002 il 02-01-2004 18:27 ]
"un uomo deve migliorare di qualcosa il mondo, se si vuole sentire realizzato..."
"Deutschland der beste Staat!"
[url:pvcj9bic]http://www.grid.org[/url:pvcj9bic] (pc vs cancro,sars,peste)
germania2002
Messaggi: 821
Iscritto il: 01 gen 1970, 01:00
Località: Cosenza
Contatta:

Messaggio da germania2002 »

up!! vedete se la mia soluzione è giusta o la devo rifare, grazie!!!! E casomai date la soluzione voi al problema![addsig]
"un uomo deve migliorare di qualcosa il mondo, se si vuole sentire realizzato..."
"Deutschland der beste Staat!"
[url:pvcj9bic]http://www.grid.org[/url:pvcj9bic] (pc vs cancro,sars,peste)
Tamaladissa
Messaggi: 173
Iscritto il: 01 gen 1970, 01:00
Località: Piacenza

Messaggio da Tamaladissa »

Non capisco quel Va/Vb = costante. Penso che l\'esercizio intenda che la barca, per andare da una sponda all\'altra mantenga costante la velocità in direzione perpendicolare alla velocità dell\'acqua che in quella direzione è nulla.
<BR>
<BR>Quindi Va/Vb non è costante perche per Va si intende la velocità di scorrimento dell\'acqua in direzione perpendicolare all barca che come hai detto tu è maggiore al centro e nulla ai lati.
<BR>
<BR>Se intendevi qualcosa d\'altro dimmelo, perchè allora non ho capito
<BR>
<BR>Ancora una cosa, il problema non specifica il tipoo di proporzionalità tra velocità e distanza dalla riva, perchè la traiettoira cambia a seconda se è direrttamente proporzionale o più che proporzionale<BR><BR>[ Questo Messaggio è stato Modificato da: Tamaladissa il 02-01-2004 21:06 ]
germania2002
Messaggi: 821
Iscritto il: 01 gen 1970, 01:00
Località: Cosenza
Contatta:

Messaggio da germania2002 »

io avevo inseso, se leggi il proble,a la barca ha una velocità Vb relativa all\'acqua, ed è costante, quindi se è proporzionale (io ho pensato direttamente) se aumenta la velocità dell\'acqua aumenta la velocità della barca. Vb/Va= kost[addsig]
"un uomo deve migliorare di qualcosa il mondo, se si vuole sentire realizzato..."
"Deutschland der beste Staat!"
[url:pvcj9bic]http://www.grid.org[/url:pvcj9bic] (pc vs cancro,sars,peste)
Tamaladissa
Messaggi: 173
Iscritto il: 01 gen 1970, 01:00
Località: Piacenza

Messaggio da Tamaladissa »

Magari poi provo a fare il problema ma tu non tieni conto delle componenti. la velocità che la barca impiega per andare da una sponda all\'altra è indipendente dalla velocità dell\'acqua.
<BR>Il rapporto tra le velocità non è utile all\'esercizio.
germania2002
Messaggi: 821
Iscritto il: 01 gen 1970, 01:00
Località: Cosenza
Contatta:

Messaggio da germania2002 »

come è indipendente, se dice
<BR><!-- BBCode Quote Start --><TABLE BORDER=0 ALIGN=CENTER WIDTH=85%><TR><TD><font size=-1>Quote:</font><HR></TD></TR><TR><TD><FONT SIZE=-1><BLOCKQUOTE>Un\'imbarcazione si stà muovendo da una sponda all\'altra con una velocità Vbarca (Vb), <!-- BBCode Start --><B>relativa all\'acqua</B><!-- BBCode End --> che è costante e perpendicolare alla corrente fluviale. </BLOCKQUOTE></FONT></TD></TR><TR><TD><HR></TD></TR></TABLE><!-- BBCode Quote End -->
<BR>
<BR>avevo pensato anch\'io che poichè Vb è perpendicolare a Va0 forse non và come soluzione, ma visto che c\'è uno spostamento è l\'unica idea che mi viene in mente, e cmq se trovo tempo (dovrei studiare ma mi scoccia...
<BR> <IMG SRC="images/forum/icons/icon_confused.gif"> ) devo anche provare il problema 1 con l\'idea di Marco (ma_go)[addsig]
"un uomo deve migliorare di qualcosa il mondo, se si vuole sentire realizzato..."
"Deutschland der beste Staat!"
[url:pvcj9bic]http://www.grid.org[/url:pvcj9bic] (pc vs cancro,sars,peste)
Tamaladissa
Messaggi: 173
Iscritto il: 01 gen 1970, 01:00
Località: Piacenza

Messaggio da Tamaladissa »

Già capisco... Quel che volevo dire è che la vel della barca varia ma nella direzione della corrente. il tempo per andare dall\'altra parte non cambia.
<BR>
<BR>Allora si considera direttamente proporzionale? Adesso ci si prova...
<BR>
<BR>Ma la soluzione ce l\'hai? Al max vediamo se abbiam considerato giusto
Bloccato